Đến nội dung

Nguyenhuyen_AG

Nguyenhuyen_AG

Đăng ký: 09-09-2010
Offline Đăng nhập: 10-01-2019 - 16:22
****-

#475510 [VMO 2014] Ngày 2 - Bài 6 - Đại số

Gửi bởi Nguyenhuyen_AG trong 05-01-2014 - 16:49

ở đoạn: $4x^2y^2z^2[(x^2+y^2)(xy+z^2)]=4x^2y^2z^2[2x^2y^2+z^2(x^2+y^2)]$

đâu có được đâu?

Cài này đúng là $4x^2y^2z^2[(x^2+y^2)(xy+z^2)] \ge 4x^2y^2z^2[2x^2y^2+z^2(x^2+y^2)]$ theo bất đẳng thức AM-GM, mình viết nhầm đấu $\ge$ thành $=.$




#475425 Cho $a;b;c$ tm: $a+b+c=1$. Tìm Max $P=(a^{3...

Gửi bởi Nguyenhuyen_AG trong 05-01-2014 - 10:41

Điều kiện của $a,\;b,\;c$ là gì vậy ?




#475423 [VMO 2014] Ngày 2 - Bài 6 - Đại số

Gửi bởi Nguyenhuyen_AG trong 05-01-2014 - 10:32

 

Bài 6 (7 điểm). Tìm giá trị lớn nhất của biểu thức
$$T=\frac{x^3y^4z^3}{(x^4+y^4)(xy+z^2)^3}+\frac{y^3z^4x^3}{(y^4+z^4)(yz+x^2)^3}+\frac{z^3x^4y^3}{(z^4+x^4)(zx+y^2)^3}$$
với $x,y,z$ là các số thực dương.

 
 
Theo bất đẳng thức AM-GM, ta có
\[(xy+z^2)^2\ge 4xy\cdot z^2.\]
\[x^4+y^4 \ge \frac{(x^2+y^2)^2}{2} \ge 2xy\cdot \frac{x^2+y^2}{2}=xy(x^2+y^2).\]
 
Từ đó, suy ra 
\[(x^4+y^4)(xy+z^2)^3 \ge 4xyz^2 \cdot xy(x^2+y^2)\cdot (xy+z^2)\ge 4x^2y^2z^2[2x^2y^2+z^2(x^2+y^2)],\]
hay là
\[\frac{{{x}^{3}}{{y}^{4}}{{z}^{3}}}{({{x}^{4}}+{{y}^{4}}){{(xy+{{z}^{2}})}^{3}}} \le \frac{xy^2z}{2x^2y^2+z^2(x^2+y^2)}.\]
Đặt $a=xy,\;b=yz,\;c=zx.$ Ta được
\[P \le \frac{1}{4}\left (\frac{ab}{2a^2+b^2+c^2}+\frac{bc}{2b^2+c^2+a^2} + \frac{ca}{2c^2+a^2+b^2} \right ).\]
Theo bất đẳng thức AM-GM, thì
\[\frac{ab}{2a^2+b^2+c^2} \le \frac{ab}{\sqrt{2a^2(b^2+c^2)}}=\frac{1}{\sqrt{2}}\cdot \sqrt{\frac{b^2}{b^2+c^2}}.\]
Nên 
\[P\le \frac{1}{4\sqrt{2}}\left ( \sqrt{\frac{a^2}{a^2+b^2}}+\sqrt{\frac{b^2}{b^2+c^2}}+\sqrt{\frac{c^2}{c^2+a^2}} \right ).\]
Mặc khác, ta có một kết quả quen thuộc 
\[\sqrt{\frac{a^2}{a^2+b^2}}+\sqrt{\frac{b^2}{b^2+c^2}}+\sqrt{\frac{c^2}{c^2+a^2}} \le \frac{3}{\sqrt{2}}.\]
Nên $P \le \dfrac{3}{16}.$ Ngoài ra nếu $a=b=c$ hay $x=y=z,$  thì đẳng thức xảy ra. Điều này cho phép ta kết luận $P_{\max}=\dfrac{3}{16}.$
 
Nhận xét 1: Ngoài ra chúng ta có thể chứng minh 
\[\frac{ab}{2a^2+b^2+c^2}+\frac{bc}{2b^2+c^2+a^2} + \frac{ca}{2c^2+a^2+b^2} \le \frac{3}{4},\]
bằng cách đưa nó về dạng đối xứng
\[\frac{ab}{a^2+b^2+2c^2}+\frac{bc}{b^2+c^2+2a^2} + \frac{ca}{2c^2+a^2+2b^2} \le \frac{3}{4},\]
rồi sử dụng bất đẳng thức Cauchy-Schwarz dạng cộng mẫu. 
 
Ở đây mình có thêm một lời giải bằng khác. Sử dụng phân tích 
$$\frac{2ab}{2a^2+b^2+c^2} =1-\frac{(a-b)^2+a^2+c^2}{2a^2+b^2+c^2},$$
ta viết bất đẳng thức trên lại như sau 
\[ \sum \frac{(a-b)^2}{2a^2+b^2+c^2} +\sum \frac{a^2}{2a^2+b^2+c^2}+\sum \frac{c^2}{2a^2+b^2+c^2} \ge \frac{3}{2}.\] 
Không mất tính tổng quát của bài toán, ta giả sử $b$ là số ở giữa $a$ và $c.$  Khi đó theo bất đẳng thức Cauchy-Schwarz inequality, ta có
\[\sum \frac{(a-b)^2}{2a^2+b^2+c^2} \ge \frac{\left[ (a-b)+(b-c)+(a-c)\right]^2}{(2a^2+b^2+c^2)+(2b^2+c^2+a^2)+(2c^2+a^2+b^2)}, \] \[\sum \frac{a^2}{2a^2+b^2+c^2} \ge \frac{(a+b+c)^2}{(2a^2+b^2+c^2)+(2b^2+c^2+a^2)+(2c^2+a^2+b^2)}, \]
và 
\[ \sum \frac{c^2}{2a^2+b^2+c^2} \ge \frac{(c+a+b)^2}{(2a^2+b^2+c^2)+(2b^2+c^2+a^2)+(2c^2+a^2+b^2)}.\] 
Như vậy, ta cần chứng minh được 
\[\frac{4(a-c)^2+2(a+b+c)^2}{4(a^2+b^2+c^2)} \ge \frac{3}{2},\]
tương đương với.
$$(a-b)(b-c) \ge 0.$$
Bất đẳng thức cuối cùng đúng theo giả thiết của $b.$ Nên ta có điều phải chứng minh.
 
Nhận xét 2: Mình sẽ bàn một tí về xuất sứ của bài toán này (theo mình dự đoán) và những bài toán có hình thức "xấu xí" như vậy đã từng xuất hiện trước đó của kỳ thi. Thực sự thì đây là một bài toán quá dở của ngày thi thứ 2 nói riêng và của cả kỳ thi nói chung. Một bài toán cho đủ đội hình, mang tính đánh đố hơn là một bài toán của kỳ thi HSG quốc gia vì từ một bài toán rất đẹp, tác giả đã tìm cách biến đổi và sử dụng các đánh giá của mình để có được bài toán cồng kềnh như vậy. Sự xất hiện của quá nhiều $x,\;y,\;z$ làm bài toán trông không được đẹp đồng thời lũy thừa của biến quá cao làm nhiều bạn học sinh bị hoảng, vì thực tế thấy rằng các bạn học sinh thường tỏ ra bối khi đối với diện những bài toán có lũy thừa lớn hoặc bậc quá của bất đẳng thức quá lớn.
 
Ý tưởng giải bài toán quá rõ ràng, tử số có dạng tích còn mẫu số lại là tổng của các tích thì việc sử dụng bất đẳng thức AM-GM đã hiện ra ngay đó. Lại nói về việc ý tưởng của bài toán lỗ rõ ra trong bài, mình xin đề ra đây bài toán của kỳ thi TST năm 2010, một bài toán với ý tưởng tương tự
 
Bài toán 1 (Việt Nam TST 2010): Cho ba số dương $a,\;b,\;c$ thỏa mãn $16(a+b+c)\geq \frac{1}{a}+\frac{1}{b}+\frac{1}{c}.$ Chứng minh rằng
\[\sum_{cyc} \left( \frac{1}{a+b+\sqrt{2a+2c}}\right)^{3}\leq   \frac{8}{9}.\]
 
Còn về bài toán số 6 của kỳ thi, mình đoán rằng tác giả bài toán đã đi từ bất đẳng thức sau của anh Trần Quốc Anh rồi chế biến lại thành bài toán số 6 này
 
Bài toán 2 (Trần Quốc Anh): Cho $a,\;b,\;c$ là ba số thực dương thỏa mãn $abc=1.$ Chứng minh rằng
\[\displaystyle \frac{1}{(a+1)^2(b+c)}+\frac{1}{(b+1)^2(c+a)} + \frac{1}{(c+1)^2(a+b)} \le \frac{3}{8}.\]
Bất đẳng thức này có hình thức rất đẹp, tuy nhiên nếu ta viết nó lại dưới dạng thuần nhất thì nó trông rất xấu xí, như sau
\[\sum \frac{xy^3z^3}{(x^2+yz)^2(y^3+z^3)} \le \frac{3}{8}.\]
Rõ ràng thì hai bài toán quá giống nhau, và cả hai đều có thể quy về chứng minh bất đẳng thức mạnh hơn
\[\sum \sqrt{\frac{a}{a+b}} \le \frac{3}{\sqrt{2}},\]
là một bất đẳng thức quen thuộc từng là bài thi của Trung Quốc năm 2005 và đề thi Olympic 30/4 năm (năm 2009 hay 2010 mình không nhớ rõ).
 
Việc những bài toán như vầy xuât hiện càng nhiều trong đề thi cho chúng ta thấy sự thiếu ý tưởng từ khâu ra đề. Và việc này cần được khắc phục nhanh chóng để trả lại những bài toán hay, đẹp lại cho đề thi.



#474618 Bạn đã tìm lời giải như thế nào ?

Gửi bởi Nguyenhuyen_AG trong 01-01-2014 - 22:03

Mọi người tiếp tục thảo luận nhé. Lúc nãy qua bên Mathscope, thấy có một bạn post bài này.

 

Bài 3. Cho $a,\;b,\;c$ là các số thực thay đổi tùy ý thỏa mãn $ab+bc+ca=1.$ Hãy tìm giá trị nhỏ nhất của biểu thức sau

\[P=a^2+2b^2+5c^2.\]




#474480 Bạn đã tìm lời giải như thế nào ?

Gửi bởi Nguyenhuyen_AG trong 01-01-2014 - 16:32

 



Spoiler

Bài 2:  Cho các số thực không âm $a,b,c.$ Chứng minh

$$a^3+b^3+c^3-a^2b-b^2c-c^2a \ge 3(a-b)(b-c)(c-a)$$

 

 

Bài này mình giải có phần hơi thiên về kinh nghiệm một tí. Vì mình thấy những bài toán dạng đa thức như vậy thì kỹ thuật Schur - SOS là một giải pháp tương đối tốt. Tuy nhiên đối với bài toán này thì mình chỉ mô phỏng theo kỹ thuật Schur - SOS chú không hoàn toàn dùng nó.

 

Với nhận định đó, mình viết bất đẳng thức trên lại như sau

\[a^3+b^3+c^3+2(a^2b+b^2c+c^2a)\ge 3(ab^2+bc^2+ca^2),\]

hay là

\[(a^3+b^3+c^3-3abc)+2(a^2b+b^2c+c^2a-3abc)\ge 3(ab^2+bc^2+ca^2-3abc).\]

Ta có các phân tích sau

\[a^3+b^3+c^3-3abc=(a+b+c)(a-b)^2+(a+b+c)(a-c)(b-c).\]

\[a^2b+b^2c+c^2a-3abc=c(a-b)^2+a(a-c)(b-c).\]

\[ab^2+bc^2+ca^2-3abc=c(a-b)^2+b(a-c)(b-c).\]

Từ đó có thể viết bất đẳng thức cần chứng minh lại dưới dạng

$$(a+b)(a-b)^2+(3a-2b+c)(a-c)(b-c) \ge 0.$$

Lưu ý là từ đầu đến giờ chúng ta vẫn chưa sắp thứ tự các biến hoặc chọn phần tử cực hạn. Nên chỉ có là đại lượng $(a+b)(a-b)^2$ là hiển nhiên không âm, nên để chứng minh được bài toán thì ta cần có $(3a-2b+c)(a-c)(b-c)$ cũng không âm.

 

Ta thấy, chỉ cần $a \ge b$ còn $c$ tùy ý là số lớn nhất hay nhỏ nhất thì bài toán được chứng minh.Tuy nhiên ta không thể giả sử được điều này vì bất đẳng thức đã cho chỉ có tính hoán vị chứ không đối xứng. Nên ý tưởng giả sử $c$ là số nhỏ nhất hay số lớn nhất trong hướng đi này hơi khó cho chúng ta. Như vậy còn trường hợp $c$ là số ở giữa hai số còn lại thì sao ? 

 

Nếu ta tách $a-b=(a-c)+(c-b)$ và sau đó áp dụng bất đẳng thức AM-GM sẽ ta sẽ tạo ra được $(a-c)(c-b)$ sẽ có thể giản ước cho đại lượng $(a-c)(b-c)$ và đưa bài toán ba biến bậc 3 về thành ba biến bậc nhất.

 

Với ý tưởng này, ta giả sử $c$ là số nằm giữa $a$ và $b.$ Khi đó theo bất đẳng thức AM-GM, ta có

$$(a-b)=[(a-c)+(c-b)]^2 \ge 4(a-c)(c-b)\ge 0.$$

Vậy, ta cần chứng minh

\[4(a+b)(a-c)(c-b)+(3a-2b+c)(a-c)(b-c)\ge0.\]

hay là

\[(a-c)(c-b)(a+6b-c) \ge 0.\]

Do $(a-c)(c-b)\ge0$ nên ta chỉ cần chứng minh $a+6b-c\ge0.$ Ta thấy bất đẳng thức này tương đương với

\[(a-c)+6b\ge0,\]

\[(b-c)+a+5b \ge 0.\]

Vì $c$ là số ở giữa $a$ và $b$ nên một trong hai bất đẳng thức trên đúng. Ta có điều phải chứng minh

 

Nhận xét: Thông thường khi gặp các bài toán hoán vị thì ta thường đưa chúng về dạng đối xứng vì bất đẳng thức đối xứng thường dễ xử hơn bất đẳng thức hoán vị. Liệu ý tưởng này có áp dụng được cho bài toán của bạn Ispectorgadget không ?




#474351 $$\frac{1}{a}+\frac{1}...

Gửi bởi Nguyenhuyen_AG trong 01-01-2014 - 01:53

 

Cho $a,b,c$ là các số dương thỏa mãn $abc=1$.Chứng minh
$$\frac{1}{a}+\frac{1}{b}+\frac{1}{c}+\frac{6}{a+b+c}\geq 5$$

Vừa tìm được lời giải khá thú vị cho bài này. 

 

Viết bất đẳng thức lại dưới dạng thuần nhất là

\[\frac{bc}{a^2}+\frac{ca}{b^2}+\frac{ab}{c^2}+\frac{6abc}{a^3+b^3+c^3}\ge 5,\]

hoặc

\[ \dfrac{a^3b^3+b^3c^3+c^3a^3-3a^2b^2c^2}{a^2b^2c^2} \geq \dfrac{2(a^3+b^3+c^3-3abc)}{a^3+b^3+c^3} .\] Giả sử $c\ge b \ge a.$ Ta có đánh giá \[\begin{aligned} a^3b^3+b^3c^3+c^3a^3-3a^2b^2c^2&=(ab+bc+ca)[c^2(a-b)^2+ab(a-c)(b-c)]\\&\ge (ab+bc+ca)[ab(a-b)^2+ab (a-c)(b-c)]\\&\ge (ab+bc+ca)\cdot ab \cdot [(a-b)^2+(a-c)(b-c)],\end{aligned}\] và \[a^3+b^3+c^3-3abc=(a+b+c)[(a-b)^2+(a-c)(b-c)].\] Do đó, ta cần chứng minh \[\frac{(ab+bc+ca)[(a-b)^2+(a-c)(b-c)]}{abc^2}\ge \frac{2(a+b+c)[(a-b)^2+(a-c)(b-c)]}{a^3+b^3+c^3},\] tương đương với \[(ab+bc+ca)(a^3+b^3+c^3)\ge 2abc^2(a+b+c).\] Theo bất đẳng thức AM-GM, thì \[abc(a+b+c)\le \frac{(ab+bc+ca)^2}{3}.\] Do đó, ta cần chứng minh được \[3(a^3+b^3+c^3)\ge 2c(ab+bc+ca),\] hay là \[3(a^3+b^3+c^3)\ge 2c^2(a+b)+2abc.\] Vì \[a^3+b^3\ge \frac{(a+b)^3}{4}\;\;,\text{và}\;\;c^3=\frac{c^3}{2} +\frac{c^3}{2}.\] Nên theo bất đẳng thức AM-GM, ta có \[a^3+b^3+c^3\ge 3\sqrt[3]{\frac{(a+b)^3}{4}\cdot\frac{c^3}{2}\cdot\frac{c^3}{2}}=\frac{3}{2\sqrt[3]{2}}\cdot c^2(a+b).\] Từ đó suy ra \[3(a^3+b^3+c^3)\ge\frac{9}{2\sqrt[3]{2}}\cdot c^2(a+b).\] Do đó, ta cần chứng minh \[\frac{9}{2\sqrt[3]{2}}\cdot c^2(a+b)\ge 2c^2(a+b)+2abc,\] \[\left (\frac{9}{2\sqrt[3]{2}} -2 \right )c(a+b)\ge 2ab.\] Bất đẳng thức này đúng vì $c(a+b)\ge c\cdot 2a \ge 2ab,$ và $\left (\dfrac{9}{2\sqrt[3]{2}} -2 \right )>1.$

 

Sử dụng phương pháp tương tự, ta cũng chứng minh được bất đẳng thức Việt Nam TST 2013 :D

$$ \dfrac{bc}{a^2}+\dfrac{ca}{b^2}+\dfrac{ab}{c^2} + \dfrac{13abc}{a^3+b^3+c^3+abc} \geq \dfrac{25}{4}.$$

với cùng điều kiện.




#474345 Bạn đã tìm lời giải như thế nào ?

Gửi bởi Nguyenhuyen_AG trong 01-01-2014 - 01:15

Trước hết mình xin chúc các thành viên của Diễn Đàn Toán Học năm mới vui vẻ và hạnh phúc, đạt được nhiều thành công trong công việc cũng như học tập và ... tìm gấu  :lol:

 

Bây giờ mình sẽ vào vấn đề chính. Như chúng ta đã thấy đa phần những sách tham khảo hiện nay, đều viết theo kiểu thị trường "nêu phương pháp rồi giải" chứ không làm được một điều, đó là giải thích cho bạn đọc hiểu được cặn kẽ cội nguồn của ý tưởng: Quan sát bài toán thì thấy gì, định hướng ra làm sao, ta nên làm gì với nó, ...
 
Chính điều đó đã dẫn đến một hệ lụy là, khi gặp những "dạng mới" thì học sinh bị bỡ ngỡ (do suốt ngày chỉ học "mấy dạng" trong các sách kia thôi mà) và thu được những kết quả không tốt. Vì những nguyên nhân như vậy, nay mình xin mở topic này, nêu ra các ví dụ để mọi người cùng nhau thảo luận và phân tích cho bạn đọc biết thế nào là một cách học đúng (không máy móc). Vì vậy rất mong nhận được sự nhiệt tình từ phía các thành viên của diễn đàn.
 
Tất nhiên là không phải bài toán nào cũng có thể nêu ra được ý tưởng nó còn phụ thuộc vào nhiều ý tố mà đa phần là do kinh nghiệm của từng người. Chẳng hạn như nếu đưa ra bài toán sau
Cho $a,\;b,\;c$ là ba số thực tùy ý. Hãy chứng minh
\[(a^2+b^2+c^2)^2 \ge 3(a^3b+b^3c+c^3a),\]
mà yêu cầu chúng ta nêu ra ý tưởng để giải thì đúng là éo le vì thậm chí một lời giải tự nhiên cho bài toán này vẫn chưa được tìm ra thì làm sao có thể cho ý tưởng để giải được.
 
Chốt, topic này chúng ta không chú trọng việc giải! Mà chú trọng vào việc phân tích và bình luận. Và hy vọng rằng mọi người sẽ chia sẻ bằng tất cả lòng nhiệt huyết, không có tình trạng giấu nghề.

 

Mình xin bắt đầu bằng bài toán sau:
 

Bài toán 1. Với $a,\;b$ là hai số thực dương tùy ý. Hãy chứng minh bất đẳng thức 

\[\left( {{a}^{2}}+b+\frac{3}{4} \right)\left( {{b}^{2}}+a+\frac{3}{4} \right)\ge \left( 2a+\frac{1}{2} \right)\left( 2b+\frac{1}{2} \right).\]

Lời giải. Quan sát một chút ta thấy mỗi biểu thức trong ngoặc ở vế phải có thể phân tách thành tổng của hai số hạng mà một trong nó sẽ giống với một thừa số bên vế phải, cụ thể là

\[2\left( {{a}^{2}}+b+\frac{3}{4} \right)=\left( 2{{a}^{2}}+1 \right)+\left( 2b+\frac{1}{2} \right).\]

Mặt khác với $a=b=\frac{1}{2}$ thì đẳng thức của bài toán xảy ra, đồng thời $2{{a}^{2}}+1=2b+\frac{1}{2},$ vì thế nếu sử dụng đẳng thức này sau đó sử dụng tiếp bất đẳng thức AM-GM cho vế trái thì ta có thể giản ước bớt đi đại lượng $\left( 2a+\frac{1}{2} \right)\left( 2b+\frac{1}{2} \right)$ cho vế phải mà vẫn đảm bảo được dấu bằng của bài toán.

 

Ý tưởng là như vậy và ta sẽ tiến hành như sau. Viết bất đẳng thức bất đẳng thức cần chứng minh lại dưới dạng

\[\left[ \left( 2{{a}^{2}}+1 \right)+\left( 2b+\frac{1}{2} \right) \right]\left[ \left( 2{{b}^{2}}+1 \right)+\left( 2a+\frac{1}{2} \right) \right]\ge 4\left( 2a+\frac{1}{2} \right)\left( 2b+\frac{1}{2} \right).\]

Sử dụng bất đẳng thức AM-GM, ta có

$$\left( 2{{a}^{2}}+1 \right)+\left( 2b+\frac{1}{2} \right)\ge 2\sqrt{( 2{{a}^{2}}+1)\left( 2b+\frac{1}{2} \right)},$$

\[\left( 2{{b}^{2}}+1 \right)+\left( 2a+\frac{1}{2} \right)\ge 2\sqrt{\left( 2{{b}^{2}}+1 \right)\left( 2a+\frac{1}{2} \right)}.\]

Từ đánh giá trên ta đưa bài toán về chứng minh

\[\left( 2{{a}^{2}}+1 \right)\left( 2{{b}^{2}}+1 \right)\ge \left( 2a+\frac{1}{2} \right)\left( 2b+\frac{1}{2} \right).\]

Bất đẳng thức cuối cùng được suy ra từ đánh giá cơ bản sau đây

\[2{{x}^{2}}+1\ge 2x+\frac{1}{2},\]

với $x$ là số thực bất kỳ.

 

Nhưng điều này là hiển nhiên vì

\[2{{x}^{2}}+1-\left( 2x+\frac{1}{2} \right)=2{{\left( x-\frac{1}{2} \right)}^{2}}\ge 0. \]

Bài toán được chứng minh.

 

Trên đây là ý tưởng của mình để giải bài toán này. Nó khá dễ nên cách chứng minh của nó cũng dễ tìm ra, nhưng điều mình muốn nói ở đây là ý tưởng. Những ý tưởng trên có thể chưa phải là hay nhất, nhưng cũng từ những suy luận có lý mà ra. :D Các bạn hãy thử đề xuất bài toán (có thể nêu được ý tưởng giải) để mọi người cùng bàn luận nhé. 

 

À, không biết bài toán trên còn hướng tiếp cận nào khác không nhỉ ?




#473713 $A=\sum \frac{a^2}{3a^2+(b+c)^2}$

Gửi bởi Nguyenhuyen_AG trong 29-12-2013 - 16:08



Cho $a,b,c\geq 0.$ Tìm giá trị lớn nhất của :

   $A=\frac{a^2}{3a^2+(b+c)^2}+\frac{b^2}{3b^2+(c+a)^2}+\frac{c^2}{3c^2+(a+b)^2}$

Giả sử các đại lượng trong $A$ đều xác định. Theo bất đẳng thức AM-GM ta có

\[\frac{a^2}{3a^2+(b+c)^2}=\frac{a^2}{2a^2+[a^2+(b+c)^2]}\le \frac{a^2}{2a^2+2a(b+c)}=\frac{1}{2}\cdot\frac{a}{a+b+c}.\]

Nên 

\[\sum \frac{a^2}{3a^2+(b+c)^2}\le \frac{1}{2}\left ( \frac{a}{a+b+c}+\frac{b}{a+b+c}+\frac{c}{a+b+c} \right )=\frac{1}{2}.\]

Dễ thấy nếu có một số bằng $0$ hai số còn lại bằng nhau và lớn hơn $0$ thì đẳng thức xảy ra. Điều này cho phép ta kết luận giá trị nhỏ nhất cần tìm là $1/2.$




#473583 Cho x,y,u,v>0.CMR: $\frac{xy+xv+uy+uv}{x+y+u+v...

Gửi bởi Nguyenhuyen_AG trong 29-12-2013 - 00:31

Cho x,y,u,v>0.CMR:

$\frac{xy+xv+uy+uv}{x+y+u+v} \geq \frac{xy}{x+y}+\frac{uv}{u+v}$

 

Bất đẳng thức cần chứng minh tương đương với

\[\frac{(y+v)(x+u)}{x+y+u+v} \geq \frac{xy}{x+y}+\frac{uv}{u+v},\]

\[\left (x+y-\frac{4xy}{x+y}  \right )+\left (u+v-\frac{4uv}{u+v}  \right ) \geq x+y+u+v -\frac{4(y+v)(x+u)}{x+y+u+v},\]

\[\frac{(x-y)^2}{x+y}+\frac{(u-v)^2}{u+v}\geq \frac{(x+u-y-v)^2}{x+y+u+v}.\]

Bất đẳng thức cuối cùng đúng theo bất đẳng thức Cauchy-Schwarz.




#470464 $4(xy+yz+zx)(x^3+y^3+z^3+xyz) \ge 13xyz^2(x+y+z).$

Gửi bởi Nguyenhuyen_AG trong 12-12-2013 - 11:34

Với ba số dương $x,\;y,\;z$ thỏa mãn $z=\max \{x,\;y,\;z\}.$ Chứng minh rằng $$4(xy+yz+zx)(x^3+y^3+z^3+xyz) \ge 13xyz^2(x+y+z).$$

 




#403464 Tuyển tập các BĐT tiêu biểu từ các cuộc thi chọn quốc gia các nước năm 2007-b...

Gửi bởi Nguyenhuyen_AG trong 09-03-2013 - 22:00

Có bạn nào có file tài liệu của batdangthuc.net trong đó có các bài toán trong mục "Pro of the day" của batdangthuc.net khi đó không ? Đây là một tài liệu khá hay, lúc trước mình có file đó nhưng chẳng may bị mất, giờ tìm lại không thấy nữa.


#403108 Tổng hợp các phương pháp chứng minh bất đẳng thức

Gửi bởi Nguyenhuyen_AG trong 08-03-2013 - 22:48

Đây là file gốc "YẾU TỐ ÍT NHẤT" mình xin của anh Cẩn, tác giả của bài viết này. Mình chia sẻ cho mọi người cùng tham khảo. :D

21.Phương pháp "Yếu tố ít nhất" [Võ Quốc Bá Cẩn] :

File gửi kèm




#399819 Bất đẳng thức Việt Nam TST 2006

Gửi bởi Nguyenhuyen_AG trong 24-02-2013 - 21:51

Trong kỳ thi chọn đội tuyển Việt Nam dự thi IMO 2006 trong bài thi ngày thứ 2 có bài toán bất đẳng thức sau đây của thầy Trần Nam Dũng
Chứng minh rằng nếu $x,y,z$ thuộc $[1,2]$ thì $$(x+y+z)\left ( \frac{1}{x} +\frac{1}{y}+\frac{1}{z}\right ) \ge 6\left ( \frac{x}{y+z}+\frac{y}{z+x}+\frac{z}{x+y} \right ).$$
bài toán còn đúng trong điều kiện mạnh hơn khi $x,y,z$ là độ dài ba cạnh của tam giác, tức là ta có thể đưa bài toán về điều kiện ba biến dương như sau $$(a+b+c)\left ( \frac{1}{a+b} +\frac{1}{b+c}+\frac{1}{c+a}\right ) \ge 3\left ( \frac{a+b}{b+2c+a}+\frac{b+c}{c+2a+b}+\frac{c+a}{a+2b+c} \right ).$$ Theo mình được biết thì bài toán có hai lời giải là S.O.S và dồn biến, hiện vẫn chưa có lời giải bằng cổ điển cho bài toán này, mình vẫn đang tìm một lời giải cổ điển cho bài toán này nhưng vẫn chưa tìm được, nay viết lên đây hi vọng nhận được sự quan tâm của các bạn. :D


#391177 Hỏi về Header trong LaTeX

Gửi bởi Nguyenhuyen_AG trong 28-01-2013 - 20:42

Mọi người cho mình hỏi vì sao trong file này thì từ trang thứ 7 header và đánh số trang nó không hiển thị nữa ? Lỗi này phải sửa như thế nào ? Chân thành cám ơn.
http://www.mediafire...n6j2poxbkd62ex9


#387003 Chứng minh rằng: $\sqrt{abc}(\sqrt{a}+\sqrt{b}+\sqrt...

Gửi bởi Nguyenhuyen_AG trong 15-01-2013 - 20:38

Cho a,b,c là độ dài 3 cạnh tam giác. Chứng minh rằng:
$\sqrt{abc}(\sqrt{a}+\sqrt{b}+\sqrt{c}) \geq (a-b+c)(a+b-c)+(-a+b+c)(a+b-c)+(-a+b+c)(a-b+c) $

Ta viết bất đẳng thức cần chứng minh lại như sau $$a^2+b^2+c^2+\sqrt{abc}(\sqrt{a}+\sqrt{b}+\sqrt{c}) \ge 2(ab+bc+ca).$$ Do tính thuần nhất nên ta có thể chuẩn hóa cho $abc=1.$ Khi đó theo bất đẳng thức AM-GM, ta có $$\sqrt{a}+\sqrt{b}+\sqrt{c}\ge 3 \sqrt[3]{\sqrt{abc}}=3=2abc+1.$$ Như vậy ta chỉ cần chứng minh được $$a^2+b^2+c^2+2abc+1 \ge 2(ab+bc+ca).$$ Là một kết quả quen thuộc. :D